subject
Physics, 16.07.2020 21:01 PrincessKeliah5538

A pair of long, rigid metal rods each of length L, lie parallel to each other on a perfectly smooth table. Their ends are connected by identical, very light conduction springs of force constant k each and negligible unstretched length. If the current I runs through the circuit, the springs will stretch. At what separation will the rods remain at rest? Assume that k is large enough so that the separation of rods will be much less than L. Express your answer in terms of I, L, k and appropriate constants (μo,π).

ansver
Answers: 1

Another question on Physics

question
Physics, 21.06.2019 22:00
Identify the forces that contribute to the centripetal force on the object in each of the following examples: a. a bicyclist moving around a flat, circular track b. a bicycle moving around a flat, circular track c. a racecar turning a corner on a steeply banked curve
Answers: 1
question
Physics, 21.06.2019 23:00
If an inclined plane is 5 m long and 2 m high, what is its mechanical advantage? a. 2.5 b. 3 c. 7 d. 10
Answers: 1
question
Physics, 22.06.2019 04:00
It can take some of those large , industrial vehicles up to feet to stop when traveling only 60 mph, and therefore , you shoukd be mindful to not be jn the traffic right in front of them a. 100 feet b. 200 feet c. 335 feet d. 425 feet
Answers: 1
question
Physics, 22.06.2019 04:40
How is the gravitational force related to the distance between two objects?
Answers: 1
You know the right answer?
A pair of long, rigid metal rods each of length L, lie parallel to each other on a perfectly smooth...
Questions
question
English, 19.10.2021 18:50
question
Chemistry, 19.10.2021 18:50
question
Mathematics, 19.10.2021 18:50
question
Mathematics, 19.10.2021 18:50
Questions on the website: 13722360